Difference between revisions of "2006 AMC 10A Problems/Problem 1"

m (See Also)
(dollars)
Line 1: Line 1:
 
== Problem ==
 
== Problem ==
Sandwiches at Joe's Fast Food cost $3 each and sodas cost $2 each.  How many dollars will it cost to purchase 5 sandwiches and 8 sodas?
+
Sandwiches at Joe's Fast Food cost <dollar/>3 each and sodas cost <dollar/>2 each.  How many dollars will it cost to purchase 5 sandwiches and 8 sodas?
  
 
<math> \mathrm{(A) \ } 31\qquad \mathrm{(B) \ } 32\qquad \mathrm{(C) \ } 33\qquad \mathrm{(D) \ } 34\qquad \mathrm{(E) \ } 35 </math>
 
<math> \mathrm{(A) \ } 31\qquad \mathrm{(B) \ } 32\qquad \mathrm{(C) \ } 33\qquad \mathrm{(D) \ } 34\qquad \mathrm{(E) \ } 35 </math>

Revision as of 18:00, 18 November 2007

Problem

Sandwiches at Joe's Fast Food cost <dollar/>3 each and sodas cost <dollar/>2 each. How many dollars will it cost to purchase 5 sandwiches and 8 sodas?

$\mathrm{(A) \ } 31\qquad \mathrm{(B) \ } 32\qquad \mathrm{(C) \ } 33\qquad \mathrm{(D) \ } 34\qquad \mathrm{(E) \ } 35$

Solution

The answer is $(3\cdot 5)+(2\cdot 8) = 15+16=31 \Longrightarrow \mathrm{(A)}$

See also

2006 AMC 10A (ProblemsAnswer KeyResources)
Preceded by
First Question
Followed by
Problem 2
1 2 3 4 5 6 7 8 9 10 11 12 13 14 15 16 17 18 19 20 21 22 23 24 25
All AMC 10 Problems and Solutions